funzionale su N

Polinomi, disuguaglianze, numeri complessi, ...
Rispondi
Avatar utente
what
Messaggi: 158
Iscritto il: 01 gen 1970, 01:00
Località: roma

funzionale su N

Messaggio da what »

Trovare tutte le funzioni $ f: \mathbb N \rightarrow \mathbb N $ tali che

$ xf(y)+yf(x)=(x+y)f(x^2+y^2) $

per ogni x,y naturali.

(forse potrebbe anche stare in teoria dei numeri... boh)
Avatar utente
edriv
Messaggi: 1638
Iscritto il: 16 feb 2006, 19:47
Località: Gradisca d'Isonzo
Contatta:

Messaggio da edriv »

La soluzione mi sembra troppo facile, quindi è probabile che sia sbagliata...

Per prima cosa sostituisco x=y:
$ 2xf(x)=2xf(2x^2) $
$ f(x)=f(2x^2)=f(2(2x^2)^2)=\ldots $ (1)
Questo ci servirà solo dopo, ma dice già qualcosa di importante: la funzione, se esiste, sembra abbastanza "patetica": anche per valori arbitrariamente grandi di x, legati in modo strano ai precedenti, assume un valore relativamente piccolissimo. Questo ci da la traccia della dimostrazione: cerchiamo di dimostrare che la funzione non esiste. Anzi, che è costante: f(x)=c funziona benissimo.

Poniamo $ f(1)=c $ e sostituiamo y=1:
$ cx + f(x)=(x+1)(\ldots) $
$ f(x) \equiv -cx \pmod {x+1} $, sottraggo e aggiungo c a destra:
$ f(x) \equiv c \pmod {x+1} $ (2).

Se f(x), positivo, è minore di x+1, allora chiaramente deve essere uguale a c. Ora vogliamo dimostrare che f(x)=c per ogni x.
Un elemento della successione $ x,2x^2,2(2x^2)^2,\ldots $ sarà maggiore di f(x). Chiamiamo d questo elemento.
Allora, per la (2) induttivata:
$ f(d)=f(x) $ (3)
Ma $ f(x)<d $
Quindi $ f(d) < d < d+1 $
Da cui, per la conseguenza della (2):
$ f(d)=c $
Ma da qui è ovvio concludere, per la (3):
$ f(x)=c $

q.e.d.

p.s. fonte del problema?
Avatar utente
teppic
Moderatore
Messaggi: 722
Iscritto il: 26 ago 2005, 09:50
Località: Parma
Contatta:

Messaggio da teppic »

Bene edriv. :)

Altro approccio possibile: $ f(x^2+y^2) $ è la media pesata di $ f(x) $ e $ f(y) $. Supponiamo che $ f $ non sia costante e siano $ x $ e $ y $ scelti in modo che le loro immagini $ f(x)<f(y) $ siano i due minimi valori distinti nell'immagine di $ f $. Allora si giunge subito ad un assurdo perché $ f(x^2+y^2) $ dovrebbe essere strettamente compresa tra $ f(x) $ e $ f(y) $.

Sono arrivato a questa dimostrazione dopo due ore di tentativi in cui non mi ero accorto di non stare usando un'ipotesi: c'è qualcuno che riesca a dimostrare che $ f $ è costante anche se si suppone che sia a valori in $ \mathbb R $?

[Edit] Ci sono riuscito! Comunque aspetto a postare perché è un problema molto carino.
piever
Messaggi: 645
Iscritto il: 18 feb 2006, 13:15
Località: Roma
Contatta:

Messaggio da piever »

ah scusa, in effetti mi pareva troppo facile... :oops:

Work In Progress allora...
Ultima modifica di piever il 21 set 2006, 21:16, modificato 1 volta in totale.
"Sei la Barbara della situazione!" (Tap)
Avatar utente
teppic
Moderatore
Messaggi: 722
Iscritto il: 26 ago 2005, 09:50
Località: Parma
Contatta:

Messaggio da teppic »

Eh, no, Piever! Con a valori in $ \mathbb R $ intendevo $ f:\mathbb N\rightarrow\mathbb R $, non che il dominio fossero i reali!
Avatar utente
what
Messaggi: 158
Iscritto il: 01 gen 1970, 01:00
Località: roma

Messaggio da what »

ok ad entrambi, io l'avevo fatta come teppic.

la fonte del problema non la so, nel senso che mi è stato proposto da un imo-boy lussemburghese...
Avatar utente
teppic
Moderatore
Messaggi: 722
Iscritto il: 26 ago 2005, 09:50
Località: Parma
Contatta:

Messaggio da teppic »

Visto che non ci sono state risposte, scrivo la mia dimostrazione del problema modificato togliendo l'ipotesi che la funzione sia a valori naturali.

Sia $ \alpha=f(0) $ vogliamo dimostrare che $ f(x)=\alpha $ per ogni $ x\in\mathbb N $.

Ragionando per assurdo, sia $ ~n $ il minimo $ x $ tale che $ f(x)\neq\alpha $.

Per prima cosa proviamo a supporre che $ ~n $ sia della forma $ n=ak+b $, con $ a>b\geq1, k\geq2, a\neq bk $. In questo caso posso scrivere

$ \displaystyle (ak+b)^2-(ak-b)^2=4abk=(bk+a)^2-(bk-a)^2 $

e quindi, ponendo $ m=ak-b,p=bk+a,q=bk-a $ si trova che

$ \displaystyle n^2+q^2 = m^2+p^2 $

da cui si deduce subito che

$ \noindent\displaystyle\frac q{q+n}f(n)+\frac n{q+n}f(q) = f(n^2+q^2) = f(m^2+p^2) = \\\frac p{p+m}f(m)+\frac m{p+m}f(p) $

Le condizioni imposte su $ a,b,k $ fanno sì che $ m,p,q $ sono tutti positivi e strettamente minori di $ ~n $, quindi $ f(m)=f(p)=f(q)=\alpha $ e sostituendo l'equazione si semplifica in $ qf(n) = q\alpha $; siccome $ q>0 $ si ha un assurdo.

Necessariamente quindi $ ~n $ non deve essere della forma $ n=ak+b $ con le condizioni imposte. Tuttavia, scegliendo $ k=2, b=1,2 $(a seconda della parità di $ ~n $) tutti i numeri interi maggiori o uguali a $ ~7 $ possono essere scritti in quella forma, perciò il minimo $ ~n $ dovrebbe essere minore o uguale a $ ~6 $, ma tali valori si mettono a posto a mano con delle semplici verifiche, arrivando all'assurdo finale.
Rispondi